The unified state exam in mathematics at the basic level consists of 20 tasks. Task 20 tests the skills of solving logical problems. The student should be able to apply his knowledge to solve problems in practice, including arithmetic and geometric progression. Here you can learn how to solve task 20 of the Unified State Examination in mathematics at a basic level, as well as study examples and solutions based on detailed tasks.

All tasks USE database all tasks (263) USE database task 1 (5) USE database task 2 (6) USE database task 3 (45) USE database task 4 (33) USE database task 5 (2) USE database task 6 (44) ) USE base task 7 (1) USE base task 8 (12) USE base task 10 (22) USE base task 12 (5) USE base task 13 (20) USE base task 15 (13) USE base task 19 (23) USE base task 20 (32)

Two transverse stripes are marked on the tape on different sides from the middle

On the tape, on different sides from the middle, two transverse stripes are marked: blue and red. If you cut the tape along the blue strip, then one part will be longer than the other by A cm. If you cut along the red one, then one part will be longer than the other by B cm. Find the distance from the red to the blue strip.

The task about the tape is part of the USE in mathematics of the basic level for grade 11 at number 20.

Biologists have discovered a variety of amoeba

Biologists have discovered a variety of amoeba, each of which divides into two exactly in a minute. The biologist puts an amoeba in a test tube, and exactly after N hours the test tube is completely filled with amoeba. How many minutes will it take for the whole test tube to be filled with amoebas if we put not one, but K amoebas in it?

When demonstrating summer clothes, the outfits of each fashion model

When demonstrating summer clothes, the outfits of each fashion model differ in at least one of three elements: a blouse, a skirt and shoes. In total, the fashion designer prepared for the demonstration A types of blouses, B types of skirts and C types of shoes. How many different outfits will be shown in this demo?

The task about outfits is part of the USE in mathematics of the basic level for grade 11 at number 20.

A group of tourists overcame a mountain pass

A group of tourists overcame a mountain pass. They covered the first kilometer of the ascent in K minutes, and each next kilometer covered L minutes longer than the previous one. The last kilometer before the summit was covered in M ​​minutes. After resting N minutes at the top, the tourists began their descent, which was more gentle. The first kilometer after the top was covered in P minutes, and each next one is R minutes faster than the previous one. How many hours did the group spend on the entire route if the last kilometer of the descent was covered in S minutes.

The task is part of the USE in mathematics of the basic level for grade 11 at number 20.

The doctor prescribed the patient to take the medicine according to this scheme.

The doctor prescribed the patient to take the medicine according to the following scheme: on the first day he should take K drops, and on each next day - N drops more than on the previous one. How many vials of medicine should the patient buy for the entire course of treatment if each contains M drops?

The task is part of the USE in mathematics of the basic level for grade 11 at number 20.

According to Moore's empirical law, the average number of transistors on microcircuits

According to Moore's empirical law, the average number of transistors on microcircuits increases N times every year. It is known that in 2005 the average number of transistors on a microcircuit was K million. Determine how many millions of transistors on the microcircuit were on average in 2003.

The task is part of the USE in mathematics of the basic level for grade 11 at number 20.

Oil company drilling a well to extract oil

An oil company is drilling a well for oil production, which, according to geological exploration data, lies at a depth of N km. During the working day, drillers go L meters deep, but during the night the well “silts up” again, that is, it is filled with soil for K meters. How many working days will oil workers drill a well to the depth of oil?

The task is part of the USE in mathematics of the basic level for grade 11 at number 20.

Refrigerator sales volume in a home appliances store is seasonal

In a home appliance store, sales of refrigerators are seasonal. In January, K refrigerators were sold, and in the next three months they sold L refrigerators each. Since May, sales have increased by M units compared to the previous month. Since September, the volume of sales began to decrease by N refrigerators every month relative to the previous month. How many refrigerators did the store sell in a year?

The task is part of the USE in mathematics of the basic level for grade 11 at number 20.

The coach advised Andrey to spend on the treadmill on the first day of classes

The trainer advised Andrey to spend L minutes on the treadmill on the first day of training, and to increase the time spent on the treadmill by M minutes at each next session. How many sessions will Andrey spend on the treadmill in total N hours K minutes if he follows the coach's advice?

The task is part of the USE in mathematics of the basic level for grade 11 at number 20.

Every second a bacterium divides into two new bacteria.

Every second a bacterium divides into two new bacteria. It is known that bacteria fill the entire volume of one glass in N hours. In how many seconds will the glass be filled with bacteria by 1/K part?

The task is part of the USE in mathematics of the basic level for grade 11 at number 20.

There are four gas stations on the ring road: A, B, C and D

There are four gas stations on the ring road: A, B, C and D. The distance between A and B is K km, between A and C is L km, between C and D is M km, between D and A is N km (all distances measured along the ring road along the shortest arc). Find the distance (in kilometers) between B and C.

The task about the gas station is part of the USE in mathematics of the basic level for grade 11 at number 20.

Sasha invited Petya to visit, saying that he lives

Sasha invited Petya to visit, saying that he lives in the K entrance in apartment No. M, but he forgot to say the floor. Approaching the house, Petya discovered that the house was N-storey. What floor does Sasha live on? (On all floors, the number of apartments is the same, the numbers of apartments in the building start from one.)

The task about apartments and houses is part of the USE in mathematics of the basic level for grade 11 at number 20.

Mysikova Julia

The unified state exam in mathematics at the basic level consists of 20 tasks. Task 20 tests the skills of solving logical problems. The student should be able to apply their knowledge to solve problems in practice, including arithmetic and geometric progression. In this work, we analyze in detail how to solve task 20 of the USE in mathematics at a basic level, as well as examples and methods of solutions based on detailed tasks.

Download:

Preview:

To use the preview of presentations, create a Google account (account) and sign in: https://accounts.google.com


Slides captions:

Tasks for ingenuity of the Unified State Examination in mathematics of a basic level. Tasks No. 20 Mysikova Yulia Alexandrovna, student 11 "A" of the socio-economic class Municipal educational institution "Secondary school No. 45"

Snail on a tree Solution. A snail crawls up a tree by 3 m per day, and descends by 2 m during the night. In total, it moves 3 - 2 = 1 meter per day. In 7 days it will rise by 7 meters. On the eighth day, she will crawl up another 3 meters and for the first time will be at a height of 7 + 3 = 10 (m), i.e. at the top of the tree. Answer: 8 A snail crawls up a tree by 3 m in a day, and descends by 2 m in a night. The height of a tree is 10 m. How many days will it take for a snail to crawl from the base to the top of the tree?

Gas Stations Solution. Let's draw a circle and arrange the points (gas stations) so that the distances correspond to the condition. Note that all distances between points A, C and D are known. AC=20, AD=30, CD=20. Mark point A. From point A clockwise mark point C, remember that AC=20. Now we will mark the point D, which lies at a distance of 30 from A, this distance cannot be postponed from A clockwise, since then the distance between C and D will be 10, and by condition CD = 2 0. So from A to D you need to move counterclockwise, mark point D. Since CD=20, the length of the entire circle is 20+30+20=70. Since AB=35, then point B is diametrically opposite to point A. The distance from C to B will be 35-20=15. Answer: 15. There are four gas stations on the ring road: A, B, C and D. The distance between A and B is 35 km, between A and C - 20 km, between C and D - 20 km, between D and A - 30 km (all distances are measured along the ring road in the shortest direction). Find the distance between B and C. Give your answer in kilometers.

In the cinema hall Solution. 1 way. We just count how many seats are in the rows up to the eighth: 1 - 24 2 - 26 3 - 28 4 - 30 5 - 32 6 - 34 7 - 36 8 - 38. Answer: 38. There are 24 seats in the first row of the cinema hall, and in each next row on 2 more than the previous one. How many seats are in the eighth row? 2 way. We notice that the number of places in the rows is an arithmetic progression with the first term in 24 and the difference is equal to 2. According to the formula of the nth term of the progression, we find the eighth term a 8 \u003d 24 + (8 - 1) * 2 \u003d 38. Answer: 38.

Mushrooms in a basket Solution. From the condition that among any 27 mushrooms there is at least one mushroom, it follows that the number of mushrooms is not more than 26. From the second condition, that among any 25 mushrooms there is at least one mushroom, it follows that the number of mushrooms is not more than 24. Since there are 50 mushrooms in total, then there are 24 mushrooms, and 26 milk mushrooms. Answer: 24. There are 50 mushrooms in the basket: mushrooms and milk mushrooms. It is known that among any 27 mushrooms there is at least one camelina, and among any 25 mushrooms at least one mushroom. How many mushrooms are in the basket?

Cubes in a row Solution. If we number all the cubes with numbers from one to six (not taking into account that there are cubes of different colors), then we get the total number of permutations of the cubes: Р(6)=6*5*4*3*2*1=720 Now remember that there are 2 red cubes and rearranging them (P(2)=2*1=2) will not give a new way, so the resulting product must be reduced by 2 times. Similarly, we recall that we have 3 green cubes, so we will have to reduce the resulting product by another 6 times (P (3) \u003d 3 * 2 * 1 \u003d 6) So, we get the total number of ways to arrange the cubes 60. Answer: 60 In how many ways can two identical red cubes, three identical green cubes and one blue cube be placed in a row?

On the treadmill The trainer advised Andrey to spend 15 minutes on the treadmill on the first day of training, and on each next session to increase the time spent on the treadmill by 7 minutes. How many sessions will Andrey spend on the treadmill for a total of 2 hours and 25 minutes if he follows the advice of the trainer? Solution. 1 way. We note that we need to find the sum of an arithmetic progression with the first term 15 and the difference equal to 7. According to the formula for the sum of the n first terms of the progression S n = (2a 1 + (n-1) d) * n / 2 we have 145 = (2 * 15 + (n–1)*7)*n/2, 290=(30+(n–1)*7)*n, 290=(30+7n–7)*n, 290=(23+7n)*n , 290=23n+7n 2 , 7n 2 +23n-290=0, n=5 . Answer: 5. 2 way. More labor intensive. 1-15-15 2-22-37 3-29-66 4-36-102 5-43-145. Answer: 5.

Changing coins Task 20. In the exchange office, you can perform one of two operations: for 2 gold coins, get 3 silver and one copper; for 5 silver coins, get 3 gold and one copper. Nicholas had only silver coins. After several visits to the exchange office, he had fewer silver coins, no gold coins, but 50 copper coins appeared. By how much did Nicholas's number of silver coins decrease? Solution. Let Nikolai first perform x operations of the second type, and then y operations of the first type. Then we have: Then there were 3y silver coins -5x = 90 - 100 = -10 i.e. 10 less. Answer: 10

The owner agreed the decision. It is clear from the condition that the sequence of prices for each excavated meter is an arithmetic progression with the first member a 1 = 3700 and the difference d=1700. The sum of the first n members of an arithmetic progression is calculated by the formula S n = 0.5 (2a 1 + (n - 1) d) n. Substituting the original data, we get: S 10 \u003d 0.5 (2 * 3700 + (8 - 1) * 1700) * 8 \u003d 77200. Thus, the owner will have to pay the workers 77,200 rubles. Answer: 77200. The owner agreed with the workers that they would dig a well for him on the following terms: he would pay them 3,700 rubles for the first meter, and 1,700 rubles more for each next meter than for the previous one. How much money will the owner have to pay the workers if they dig a well 8 meters deep?

Water in the pit As a result of the flood, the pit was filled with water up to a level of 2 meters. The construction pump continuously pumps out water, lowering its level by 20 cm per hour. Groundwater, on the contrary, raises the water level in the pit by 5 cm per hour. How many hours of pump operation will the water level in the pit drop to 80 cm? Solution. As a result of the operation of the pump and flooding with soil water, the water level in the pit decreases by 20-5 = 15 centimeters per hour. It takes 120:15=8 hours to lower the level by 200-80=120 centimeters. Answer: 8.

A tank with a slot In a tank with a volume of 38 liters every hour, starting at 12 o'clock, a full bucket of water with a volume of 8 liters is poured. But there is a small gap in the bottom of the tank, and 3 liters flow out of it in an hour. At what point in time (in hours) will the tank be completely filled? Solution. By the end of each hour, the volume of water in the tank increases by 8 − 3 = 5 liters. After 6 hours, that is, at 18 hours, there will be 30 liters of water in the tank. At 7 pm, 8 liters of water will be added to the tank and the volume of water in the tank will become 38 liters. Answer: 19.

Well An oil company is drilling a well for oil production, which, according to geological exploration, lies at a depth of 3 km. During the working day, drillers go 300 meters deep, but during the night the well “silts up” again, that is, it is filled with soil by 30 meters. How many working days will oil workers drill a well to the depth of oil? Solution. Given the siltation of the well, 300-30=270 meters pass during the day. This means that 2700 meters will be covered in 10 full days and another 300 meters will be covered in the 11th working day. Answer: 11.

Globe On the surface of the globe, 17 parallels and 24 meridians were drawn with a felt-tip pen. Into how many parts did the drawn lines divide the surface of the globe? Solution. One parallel divides the surface of the globe into 2 parts. Two to three parts. Three into four parts, etc. 17 parallels break the surface into 18 parts. Let's draw one meridian, and get one whole (not cut) surface. Let's draw the second meridian and we already have two parts, the third meridian will break the surface into three parts, etc. 24 meridians have divided our surface into 24 parts. We get 18*24=432. All lines will divide the surface of the globe into 432 parts. Answer: 432.

Grasshopper jumps Grasshopper jumps along the coordinate line in any direction for a unit segment per jump. How many different points on the coordinate line are there that the grasshopper can reach after making exactly 8 jumps, starting from the origin? Solution: With a little thought, we can see that the grasshopper can only end up at points with even coordinates, since the number of jumps it makes is even. For example, if he makes five jumps in one direction, then he will make three jumps in the opposite direction and end up at points 2 or −2. The maximum grasshopper can be at points, the module of which does not exceed eight. Thus, the grasshopper can end up at points: -8, -6, -4, -2, 0, 2, 4, 6 and 8; only 9 points. Answer: 9 .

New Bacteria Every second a bacterium divides into two new bacteria. It is known that the entire volume of one glass of bacteria is filled in 1 hour. How many seconds does it take for bacteria to fill half the glass? Solution. Recall that 1 hour = 3600 seconds. Every second there are twice as many bacteria. This means that it takes only 1 second to get a full glass of bacteria from a half glass of bacteria. Therefore, the glass was half filled in 3600-1=3599 seconds. Answer: 3599.

Dividing the numbers The product of ten consecutive numbers is divided by 7. What can be the remainder? Solution. The task is simple, since among ten consecutive natural numbers at least one is divisible by 7. This means that the entire product will be divisible by 7 without a remainder. That is, the remainder is 0. Answer: 0.

Where does Petya live? Task 1. The house Petya lives in has one entrance. There are six apartments on each floor. Petya lives in apartment number 50. What floor does Petya live on? Solution: We divide 50 by 6, we get the quotient 8 and 2 in the remainder. This means that Petya lives on the 9th floor. Answer: 9. Task 2. All entrances of the house have the same number of floors, and all floors have the same number of apartments. At the same time, the number of floors in the house is greater than the number of apartments per floor, the number of apartments per floor is greater than the number of entrances, and the number of entrances is more than one. How many floors are there in a building if there are 455 apartments in total? Solution: The solution to this problem follows from the decomposition of the number 455 into prime factors. 455 = 13*7*5. So the house has 13 floors, 7 apartments on each floor in the entrance, 5 entrances. Answer: 13.

Task 3. Sasha invited Petya to visit, saying that he lives in the eighth entrance in apartment No. 468, but forgot to say the floor. Approaching the house, Petya discovered that the house had twelve floors. What floor does Sasha live on? (On all floors, the number of apartments is the same, the numbers of apartments in the house start from one.) Solution: Petya can calculate that in a twelve-story building in the first seven entrances there are 12 * 7 = 84 landings. Further, sorting through the possible number of apartments on one site, you can see that there are less than six of them, since 84 * 6 \u003d 504. This is more than 468. This means that there are 5 apartments on each of the sites, then in the first seven entrances 84 * 5 \u003d 420 apartments . 468 - 420 = 48, that is, Sasha lives in apartment 48 in the 8th entrance (if the numbering was from one in each entrance). 48:5 = 9 and 3 remainder. So Sasha's apartment is on the 10th floor. Answer: 10.

Restaurant menu The restaurant menu has 6 types of salads, 3 types of first courses, 5 types of second courses and 4 types of dessert. How many salad, first, second and dessert lunch options can diners at this restaurant choose? Solution. If we number each salad, first, second, dessert, then: with 1 salad, 1 first, 1 second, one of 4 desserts can be served. 4 options. With the second second, there are also 4 options, etc. In total we get 6*3*5*4=360. Answer: 360.

Masha and the Bear The bear ate his half of the jar of jam 3 times faster than Masha, which means he still has 3 times more time left to eat cookies. Because The bear eats cookies 3 times faster than Masha and he still has 3 times more time left (he ate his half of the jam 3 times faster), then he eats 3⋅3=9 times more cookies than Masha (9 cookies are eaten by the Bear, while Masha only 1 cookie). It turns out that in a ratio of 9:1, Bear and Masha eat cookies. In total, 10 shares are obtained, which means that 1 share is equal to 160:10 \u003d 16. As a result, the Bear ate 16⋅9=144 cookies. Answer: 144 Masha and the Bear ate 160 cookies and a jar of jam, starting and finishing at the same time. At first, Masha ate jam, and the Bear ate cookies, but at some point they changed. The bear eats both three times faster than Masha. How many cookies did the Bear eat if they ate the same amount of jam?

Sticks and lines The stick has red, yellow and green transverse lines. If you cut a stick along the red lines, you get 15 pieces, if along the yellow lines - 5 pieces, and if along the green lines - 7 pieces. How many pieces will you get if you cut a stick along the lines of all three colors? Solution. If you cut a stick along red lines, you get 15 pieces, therefore, lines - 14. If you saw a stick along yellow lines - 5 pieces, therefore, lines - 4. If you saw it along green lines - 7 pieces, lines - 6. Total lines: 14+ 4 + 6 = 24 lines, therefore, there will be 25 pieces. Answer: 25

The doctor prescribed The doctor prescribed the patient to take the medicine according to the following scheme: on the first day he should take 3 drops, and on each next day - 3 drops more than on the previous one. Having taken 30 drops, he drinks 30 drops of the medicine for another 3 days, and then reduces the intake by 3 drops daily. How many vials of medicine should a patient buy for the entire course of treatment if each contains 20 ml of medicine (which is 250 drops)? Solution In the first phase of the drops, the number of drops taken per day is an increasing arithmetic progression with the first term equal to 3, the difference equal to 3 and the last term equal to 30. Therefore: Then 3 + 3(n -1)=30; 3+3n-3=30; 3n=30; n =10 , i.e. 10 days have passed according to the scheme of increasing up to 30 drops. We know the formula for the sum of arith. progressions: Calculate S10:

For the next 3 days - 30 drops each: 30 3 = 90 (drops) At the last stage of admission: I.e. 30 -3(n-1)=0; 30 -3n+3=0; -3n=-33; n=11 i.e. 11 days the medication intake decreased. Let's find the arithmetic sum. progressions 4) So, 165 + 90 + 165 = 420 drops in total 5) Then 420: 250 = 42/25 = 1 (17/25) bubbles Answer: you need to buy 2 bubbles

Household Appliance Store In a household appliance store, sales of refrigerators are seasonal. In January, 10 refrigerators were sold, and in the next three months, 10 refrigerators were sold. Since May, sales have increased by 15 units compared to the previous month. Since September, sales began to decrease by 15 refrigerators every month compared to the previous month. How many refrigerators did the store sell in a year? Solution. Let's sequentially calculate how many refrigerators were sold for each month and sum up the results: 10 4+(10+15)+(25+15)+(40+15)+(55+15)+(70-15)+ (55- 15)+(40-15)+ (25-15)== 40+25+40+55+70+55+40+25+10=120+110+130=360 Answer: 360.

Boxes Boxes of two types, having the same width and height, are stacked in a warehouse in one row 43 m long, putting them to each other in width. Boxes of one type have a length of 2m, and the other 5m. What is the smallest number of boxes required to fill the entire row without empty spaces? Solution you need to find the smallest number of boxes, then => you need to take the largest number of large boxes. So 5 7 = 35; 43 - 35 = 8 and 8:2=4; 4+7=11 So there are 11 boxes in total. Answer: 11.

Table The table has three columns and several rows. In each cell of the table, a natural number was placed so that the sum of all the numbers in the first column is 119, in the second - 125, in the third - 133, and the sum of the numbers in each row is greater than 15, but less than 18. How many rows are in the column? Solution. The total sum in all columns = 119 + 125 + 133 = 377 The numbers 18 and 15 are not included in the limit, which means: 1) if the sum in the row = 17, then the number of rows is 377: 17= =22.2 2) if the sum in the line = 16, then the number of lines is 377: 16 = = 23.5 So the number of lines = 23 (because it should be between 22.2 and 23.5) Answer: 23

Quiz and tasks The list of tasks of the quiz consisted of 36 questions. For each correct answer, the student received 5 points, for an incorrect answer, 11 points were deducted from him, and if there was no answer, they were given 0 points. How many correct answers were given by the student who scored 75 points, if it is known that he was wrong at least once? Solution. Method 1: Let X be the number of correct answers y be the number of wrong answers. Then we compose the equation 5x -11y \u003d 75, where 0

A group of tourists A group of tourists overcame a mountain pass. They covered the first kilometer of the ascent in 50 minutes, and each next kilometer passed 15 minutes longer than the previous one. The last kilometer before the summit was completed in 95 minutes. After a ten-minute rest at the top, the tourists began their descent, which was more gentle. The first kilometer after the summit was covered in an hour, and each next one is 10 minutes faster than the previous one. How many hours did the group spend on the entire route if the last kilometer of the descent was completed in 10 minutes? Solution. The group spent 290 minutes climbing the mountain, 10 minutes resting, and 210 minutes descending the mountain. In total, tourists spent 510 minutes on the entire route. Let's translate 510 minutes into hours and get that in 8.5 hours the tourists covered the entire route. Answer: 8.5

Thank you for your attention!

Task 20 Basic level of the exam

1) A snail crawls 4 m up a tree in a day, and slides 1 m in a night. The height of a tree is 13 m. In how many days will a snail crawl to the top of a tree for the first time? (4-1 \u003d 3, the morning of the 4th day will be at a height of 9m, and 4m will crawl in a day.Answer: 4 )

2) A snail crawls 4 m up a tree in a day, and slides 3 m in a night. The height of a tree is 10 m. In how many days will a snail crawl to the top of a tree for the first time? Answer: 7

3) A snail climbs 3 m up a tree in a day, and descends 2 m in a night. The height of a tree is 10 m. How many days will a snail climb to the top of a tree? Answer: 8

4) Cross lines of red, yellow and green are marked on the stick. If you cut a stick along the red lines, you get 15 pieces, if along the yellow lines - 5 pieces, and if along the green lines - 7 pieces. How many pieces do you get if you cut a stick along the lines of all three colors ? (If you cut a stick along red lines, you get 15 pieces, therefore, lines - 14. If you saw a stick along yellow lines - 5 pieces, therefore, lines - 4. If you saw it along green lines - 7 pieces, lines - 6. Total lines: 14 + 4 + 6 = 24 lines. Answer:25 )

5) On the stick are marked transverse lines of red, yellow and green. If you saw the stick along the red lines, you get 5 pieces, if along the yellow lines - 7 pieces, and if along the green lines - 11 pieces. How many pieces will you get if you cut a stick along the lines of all three colors? Answer : 21

6) Transverse lines of red, yellow and green are marked on the stick. If you cut a stick along the red lines, you get 10 pieces, if along the yellow lines - 8 pieces, if along the green lines - 8 pieces. How many pieces will you get if you cut a stick along the lines of all three colors? Answer : 24

7) In the exchange office, you can perform one of two operations:

For 2 gold coins, get 3 silver and one copper;

For 5 silver coins, get 3 gold and one copper.

Nicholas had only silver coins. After several visits to the exchange office, he had fewer silver coins, no gold coins, but 50 copper coins appeared. By how much did Nicholas's number of silver coins decrease? Answer: 10

8) At the exchange office, you can perform one of two operations:

· for 2 gold coins get 3 silver and one copper;

· For 5 silver coins, get 3 gold and one copper.

Nicholas had only silver coins. After several visits to the exchange office, he had fewer silver coins, no gold coins, but 100 copper coins appeared. By how much did Nicholas's number of silver coins decrease?? Answer: 20

9) In the exchange office, you can perform one of two operations:

1) for 3 gold coins get 4 silver and one copper;

2) for 6 silver coins, get 4 gold and one copper.

Nikola had only silver coins. After visiting the exchange office, he had fewer silver coins, no gold coins, but 35 copper coins appeared. By how much did Nikola's number of silver coins decrease? Answer: 10

10) In the exchange office, you can perform one of two operations:

1) for 3 gold coins get 4 silver and one copper;

2) for 7 silver coins, get 4 gold and one copper.

Nikola had only silver coins. After visiting the exchange office, he had fewer silver coins, no gold coins, but 42 copper coins appeared. By how much did Nikola's number of silver coins decrease? Answer: 30

11) In the exchange office, you can perform one of two operations:

1) for 4 gold coins get 5 silver and one copper;

2) for 8 silver coins, get 5 gold and one copper.

Nicholas had only silver coins. After several visits to the exchange office, he had fewer silver coins, no gold coins, but 45 copper coins appeared. By how much did Nicholas's number of silver coins decrease? Answer: 35

12) There are 50 mushrooms in the basket: mushrooms and milk mushrooms. It is known that among any 28 mushrooms there is at least one camelina, and among any 24 mushrooms at least one mushroom. How many mushrooms are in the basket? ( (50-28)+1=23 - must be redheads. (50-24)+1=27 - must be gruzdey. Answer: mushrooms in the basket 27 .)

13) There are 40 mushrooms in the basket: mushrooms and milk mushrooms. It is known that among any 17 mushrooms there is at least one camelina, and among any 25 mushrooms at least one mushroom. How many mushrooms are in the basket? ( According to the condition of the problem: (40-17)+1=24 - must be redheads. (40-25)+1=16 24 .)

14) the basket contains 30 mushrooms: mushrooms and milk mushrooms. It is known that among any 12 mushrooms there is at least one camelina, and among any 20 mushrooms at least one mushroom. How many mushrooms are in the basket? (According to the condition of the problem: (30-12)+1=19 - must be redheads. (30-20)+1=11 - must be gruzdey. Answer: saffron milk caps in a basket 19 .)

15) There are 45 mushrooms in the basket: mushrooms and milk mushrooms. It is known that among any 23 mushrooms there is at least one camelina, and among any 24 mushrooms at least one mushroom. How many mushrooms are in the basket? ( According to the condition of the problem: (45-23)+1=23 - must be redheads. (45-24)+1=22 - must be gruzdey. Answer: saffron milk caps in a basket 23 .)

16) There are 25 mushrooms in the basket: mushrooms and milk mushrooms. It is known that among any 11 mushrooms there is at least one camelina, and among any 16 mushrooms at least one mushroom. How many mushrooms are in the basket? ( Since among any 11 mushrooms at least one is a mushroom, then there are no more than 10 mushrooms. Since among any 16 mushrooms at least one is a mushroom, then there are no more than 15 mushrooms. And since there are 25 mushrooms in the basket, there are exactly 10 mushrooms, and Ryzhikov exactlyAnswer:15.

17) The owner agreed with the workers that they would dig a well for him on the following conditions: for the first meter he would pay them 4200 rubles, and for each next meter - 1300 rubles more than for the previous one. How much money will the owner have to pay the workers if they dig a well 11 meters deep ?(Answer: 117700)

18) The owner agreed with the workers that they would dig a well for him on the following conditions: for the first meter he would pay them 3,700 rubles, and for each next meter - 1,700 rubles more than for the previous one. How much money will the owner have to pay the workers if they dig a well 8 meters deep? ( 77200 )

19) The owner agreed with the workers that they dig a well on the following terms: for the first meter he will pay them 3,500 rubles, and for each next meter - 1,600 rubles more than for the previous one. How much money will the owner have to pay the workers if they dig a well 9 meters deep? ( 89100 )

20) The owner agreed with the workers that they would dig a well for him on the following conditions: for the first meter he would pay them 3,900 rubles, and for each next meter he would pay 1,200 rubles more than for the previous one. How many rubles will the owner have to pay to the workers if they dig a well 6 meters deep? (41400)

21) The trainer advised Andrey to spend 15 minutes on the treadmill on the first day of classes, and on each next lesson to increase the time spent on the treadmill by 7 minutes. How many sessions will Andrey spend on the treadmill for a total of 2 hours and 25 minutes if he follows the advice of the trainer? ( 5 )

22) The coach advised Andrey to spend 22 minutes on the treadmill on the first day of training, and on each next session to increase the time spent on the treadmill by 4 minutes until it reaches 60 minutes, and then continue to train for 60 minutes every day. In how many sessions, starting from the first one, Andrey will spend 4 hours and 48 minutes on the treadmill? ( 8 )

23) There are 24 seats in the first row of the cinema hall, and in each next row there are 2 more than in the previous one. How many seats are in the eighth row? ( 38 )

24) The doctor prescribed the patient to take the medicine according to the following scheme: on the first day he should take 3 drops, and on each next day - 3 drops more than on the previous one. Having taken 30 drops, he drinks 30 drops of the medicine for another 3 days, and then reduces the intake by 3 drops daily. How many vials of medicine should a patient buy for the entire course of treatment if each contains 20 ml of medicine (which is 250 drops)? (2) the sum of an arithmetic progression with the first term equal to 3, the difference equal to 3 and the last term equal to 30.; 165 + 90 + 135 = 390 drops; 3+ 3(n-1)=30; n=10 and 27- 3(n-1)=3; n=9

25) The doctor prescribed the patient to take the medicine according to the following scheme: on the first day he should take 20 drops, and on each next day - 3 drops more than on the previous one. After 15 days of taking the patient takes a break of 3 days and continues to take the medicine according to the reverse scheme: on the 19th day he takes the same number of drops as on the 15th day, and then reduces the dose by 3 drops daily until the dosage becomes less than 3 drops per day. How many vials of medicine should a patient buy for the entire course of treatment if each contains 200 drops? ( 7 ) drinks 615 + 615 + 55 = 1285; 1285: 200 = 6.4

26) In a household appliance store, sales of refrigerators are seasonal. In January, 10 refrigerators were sold, and in the next three months, 10 refrigerators were sold. Since May, sales have increased by 15 units compared to the previous month. Since September, sales began to decrease by 15 refrigerators every month compared to the previous month. How many refrigerators did the store sell in a year? (360) (5*10+2*25+2*40+2*55+70=360

27) On the surface of the globe, 12 parallels and 22 meridians were drawn with a felt-tip pen. Into how many parts did the drawn lines divide the surface of the globe?

A meridian is an arc of a circle connecting the North and South Poles. A parallel is a circle lying in a plane parallel to the plane of the equator. (13 22=286)

28) On the surface of the globe, 17 parallels and 24 meridians were drawn with a felt-tip pen. Into how many parts did the drawn lines divide the surface of the globe? A meridian is an arc of a circle connecting the North and South Poles. A parallel is a circle lying in a plane parallel to the plane of the equator. (18 24 =432)

29) What is the smallest number of consecutive numbers you need to take so that their product is divisible by 7? (2) If the condition of the problem sounded like this: “What is the smallest number of consecutive numbers you need to take so that their product guaranteed divisible by 7? Then it would be necessary to take seven consecutive numbers.

30) What is the smallest number of consecutive numbers you need to take so that their product is divisible by 9? (2)

31) The product of ten consecutive numbers is divided by 7. What can be the remainder? (0) Among 10 consecutive numbers, one of them will necessarily be divisible by 7, so the product of these numbers is a multiple of seven. Therefore, the remainder when divided by 7 is zero.

32) The grasshopper jumps along the coordinate line in any direction for a unit segment per jump. How many different points on the coordinate line are there that the grasshopper can reach after making exactly 6 jumps, starting from the origin? ( the grasshopper can end up at points: -6, -4, -2, 0, 2, 4 and 6; only 7 points.)

33) The grasshopper jumps along the coordinate line in any direction for a unit segment per jump. How many different points on the coordinate line are there that the grasshopper can reach after making exactly 12 jumps, starting from the origin? ( the grasshopper can end up at points: -12, -10, -8, -6, -4, -2, 0, 2, 4, 6, 8, 10 and 12; total 13 points.)

34) The grasshopper jumps along the coordinate line in any direction for a unit segment per jump. How many different points on the coordinate line are there that the grasshopper can reach after making exactly 11 jumps, starting from the origin? (may appear at points: -11, -9, -7, -5, -3, -1, 1, 3, 5, 7, 9 and 11; 12 points in total.)

35) The grasshopper jumps along the coordinate line in any direction for a unit segment per jump. How many different points on the coordinate line are there that the grasshopper can reach after making exactly 8 jumps, starting from the origin?

Note that the grasshopper can only end up at points with even coordinates, since the number of jumps it makes is even. The maximum grasshopper can be at points, the module of which does not exceed eight. Thus, the grasshopper can end up at the points: -8, -6,-2 ; −4, 0.2 , 4, 6, 8 total 9 points.

Consider such a task plan. We have the following conditions:

Total:N

From A pieces at least 1 of another type, and from B pieces at least 1 of the first type

Then: (A-1) is the minimum quantity of the first type, and (B-1) is the second.

After we do the check: (A-1) + (B-1) \u003dN.

EXAMPLE

V

SOLUTION

So: we have 35 fish in total (perch and roach)

Consider the conditions: among any 21 fish there is at least one roach, then there is at least 1 roach in this condition, therefore (21-1) = 20 is the minimum of perches. Among any 16 fish - at least one perch, arguing similarly, (16-1) = 15 - this is the minimum of roaches. Now we check: 20 + 15 = 35, that is, we got the total number of fish, which means 20 perches and 15 roaches.

ANSWER: 15 roaches

    Quiz and number of correct answers

The list of tasks of the quiz consisted of A questions. For each correct answer, the student received a points, for an incorrect answer they were deducted from him.bpoints, and in the absence of a response, 0 points were given. How many correct answers were given by the student who scoredNpoints if it is known that at least once he was mistaken?

We know how many points he earned, we know the price of a correct and incorrect answer. Based on the fact that at least one wrong answer was given, then the number of points for correct answers should exceed the number of penalty points byNpoints. Let there be x correct answers and y incorrect answers, then:

a*x= N+ b* y

x=(N+ b* y)/a

from this equality it is clear that the number in brackets must be a multiple of a. With this in mind, we can evaluate y (it is also an integer). It should be noted that the number of correct and incorrect answers should not exceed the total number of questions.

EXAMPLE

SOLUTION:

we introduce the designations (for convenience) x - correct, y - incorrect, then

5*x=75+11*y

X=(75+11*y)/5

Since 75 is divisible by five, then 11*y must also be divisible by five. Therefore, y can take multiples of five (5, 10, 15, etc.). take the first value y=5 then x=(75+11*5)/5=26 total questions 26+5=31

Y=10 x=(75+11*10)=37 total answers 37+10= 47 (more than questions) does not fit.

So in total there were: 26 correct and 5 incorrect answers.

ANSWER: 26 correct answers

    What floor?

Sasha invited Petya to visit, saying that he lives in a stairwell in apartment No.N, and I forgot to say the floor. Approaching the house, Petya discovered that the housey-storey. What floor does Sasha live on? (On all floors, the number of apartments is the same, the numbers of apartments in the building start from one.)

SOLUTION

According to the condition of the problem, we know the apartment number, the entrance and the number of floors in the house. Based on these data, it is possible to make an estimate of the number of apartments per floor. Let x be the number of apartments per floor, then the following condition must be met:

A*y*x must be greater than or equalN

From this inequality we estimate x

To begin with, we take the minimum integer value x, let it be equal to c, and check: (a-1) * y * c is less thanN, and a*y*s is greater than or equal toN.

Having chosen the value x we ​​need, we can easily calculate the floor (c): in = (N-( a-1)* c)/ c, and in is an integer and getting a fractional value, we take the nearest integer (in a big way)

EXAMPLE

SOLUTION

Let's estimate the number of apartments on the floor: 7*7*x is greater than or equal to 462, hence x is greater than or equal to 462/(7*7)=9.42, which means the minimum x=10. We check: 6 * 7 * 10 = 420 and 7 * 7 * 10 = 490 as a result, we got that the apartment by number falls into this range. Now let's find the floor: (462-6*7*10)/10=4.2 means the boy lives on the fifth floor.

ANSWER: 5th floor

    Apartments, floors, entrances

All entrances of the house have the same number of floors, and all floors have the same number of apartments. At the same time, the number of floors in the house is greater than the number of apartments per floor, the number of apartments per floor is greater than the number of entrances, and the number of entrances is more than one. How many floors are there in a building if there are X apartments in total?

This type of task is based on the following condition: if in the house E - floors, P - entrances and K - apartments on the floor, then the total number of apartments in the house should be equal to E * P * K \u003d X. so we need to represent X as a product of three numbers not equal to 1 (according to the condition of the problem). To do this, we will decompose the number X into prime factors. Having made the decomposition and taking into account the conditions of the problem, we make a selection of the correspondence between the numbers and the conditions that are indicated in the problem.

EXAMPLE

SOLUTION

Let's represent the number 105 as a product of prime factors

105=5*7*3, now let's return to the condition of the problem: since the number of floors is the largest, it is 7, the number of apartments on the floor is 5, and the entrances are 3.

ANSWER: entrances - 7, apartments on the floor - 5, entrances - 3.

    Exchange

V

For a gold coins, get from silver and copper;

For x silver coins get in gold and with 1 copper.

Nicholas had only silver coins. After the exchange office, he had fewer silver coins, no gold coins, but C copper coins appeared. By how much did Nicholas's number of silver coins decrease?

There are two exchange schemes in the exchange point:

EXAMPLE

V The exchange office can perform one of two operations:

SOLUTION

5 gold=4 silver+1 copper

10 silver=7 gold+1 copper

since no gold coins appeared, we need an exchange scheme without gold coins. Therefore, the number of gold coins must be equal in both cases. We need to find the least common multiple of the numbers 5 and 7, and bring our gold in both cases to it:

35 gold=28 silver+7 copper

50 silver=35 gold+5 copper

as a result we get

50 silver=28 silver+12 copper

We found an exchange scheme bypassing gold coins, now we need, knowing the number of copper coins, to find how many times such an operation was performed

N=60/12=5

As a result, we get

250 silver=140 silver+60 copper

Substituting, and having received the final exchange, we will find how much silver was changed. So - the number decreased by 250-140=110

ANSWER for 110 coins

6. THE GLOBE

On the surface of the globe, the x parallels and the y meridian are drawn with a marker. Into how many parts did the drawn lines divide the surface of the globe? (a meridian is an arc of a circle connecting the North and South poles, and a parallel is the boundary of a section of a globe by a plane parallel to the plane of the equator).

SOLUTION:

Since the parallel is the boundary of the section of the globe by a plane, then one will break the globe into 2 parts, two into three parts, x into x + 1 parts

The meridian is an arc of a circle (more precisely, a semicircle) and the meridian breaks the surface into y parts, therefore the total will be (x + 1) * y parts.

EXAMPLE

After similar reasoning, we get:

(30+1)*24=744 (parts)

ANSWER: 744 parts

7. CUTTINGS

On the stick are marked transverse lines of red, yellow and green. If you cut a stick along the red lines, you get A pieces, if along the yellow lines - B pieces, and if along the green lines - From the pieces. How many pieces will you get if you cut a stick along the lines of all three colors?

SOLUTION

For the solution, we take into account that the number of pieces is 1 more than the number of cuts. Now you need to find how many lines are marked on the stick. We get red (A-1), yellow - (B-1), green - (C-1). Having found the number of lines of each color and summing them up, we get the total number of lines: (A-1) + (B-1) + (C-1). We add one to the resulting number (since the number of pieces is one more than the number of cuts), we get the number of pieces if we cut along all lines.

EXAMPLE

On the stick are marked transverse lines of red, yellow and green. If you saw the stick along the red lines, you get 7 pieces, if along the yellow lines - 13 pieces, and if along the green lines - 5 pieces. How many pieces will you get if you cut a stick along the lines of all three colors?

SOLUTION

Finding the number of lines

Reds: 7-1=6

Yellow: 13-1=12

Greens: 5-1=4

Total number of lines: 6+12+4=22

Then the number of pieces: 22+1=23

ANSWER: 23 pieces

8. COLUMNS AND ROWS

V each cell of the table was placed according to a natural number so that the sum of all the numbers in the first column is equal to C1, in the second - C2, in the third - C3, and the sum of the numbers in each row is greater than Y1, but less than Y2. How many rows are in the table?

SOLUTION

Since the numbers in the cells of the table do not change, the sum of all the numbers in the table is: C=C1+C2+C3.

Now let's pay attention to the fact that the table consists of natural numbers, which means that the sum of numbers by rows must be integers and be in the range from (Y1 + 1) to (Y2-1) (since the sum of rows is strictly limited). Now we can estimate the number of rows:

C / (Y1 + 1) - the maximum number

C / (U2-1) - the minimum amount

EXAMPLE

V The table has three columns and several rows. V

SOLUTION

Find the sum of the table

С=85+77+71=233

Let's define the limits of the sum of rows

12+1=13 – minimum

15-1=14 - maximum

Estimate the number of rows in the table

233/13=17.92 maximum

233/14=16.64 minimum

Within these limits there is only one whole number - 17

ANSWER: 17

9. REFUELING AT THE RING

and D. The distance between A and B - 35 km, between A and B - 20 km, between B and G - 20 km, between G and A and V.

SOLUTION

Having carefully read the problem, we will notice that in practice the circle is divided into three arcs AB, VG and AG. Based on this, we will find the length of the entire circle (ring). For this task, it is equal to 20+20+30=70 (km).

Now, having placed all the points on the circle and signed the lengths of the corresponding arcs, it is easy to determine the required distance. In this problem BV=AB-AB, i.e. BV=35-20=15

ANSWER: 15 km

10. COMBINATIONS

SOLUTION

To solve this type of problem, remember what a factorial is.

Factorial of a numberN! called the product of consecutive numbers from 1 toN, that is, 4!=1*2*3*4.

Now back to the task. Find the total number of cubes: 3+1+1=5. Since we have three cubes of the same color, the total number of cubes can be found using the formula 5!/3! We get (5*4*3*2*1)/(1*2*3)=5*4=20

ANSWER: 20 ways to arrange

11 . WELLS

The owner agreed with the workers that they would dig a well for him under the following conditions: for the first meter he would pay them X rubles, and for each next meter - Y rubles more than for the previous one. How many rubles will the owner have to pay to the workers if they dig a well deepNmeters?

SOLUTION:

Since the owner increases the price for each meter, he will pay for the second (X + Y), for the third - (X + 2Y), for the fourth (X + 3Y), etc. It is not difficult to see that this payment system resembles an arithmetic progression, where a1 = X,d= Y, n= N. Then

Payment for work is nothing but the sum of this progression:

S= ( (2a+d(n-1))/2) n

EXAMPLE:

SOLUTION

Based on the above, we geta1=4200

d=1300

n=11

Substituting this data into our formula, we get

S=((2*4200+1300(11-1)/2)*11=((8400+13000)/2)*11=10700*11=117700

ANSWER: 117700

12 . POSTS AND WIRES

X poles connected by wires, so that exactly Y wires extend from each. How many wires are strung between the poles?

SOLUTION

Find how many gaps between the pillars. Between two there is one gap, between three - two, between four - 3, between X - (X-1).

At each gap Y wires, then (X-1) * Y is the total wires between the poles.

EXAMPLE

The ten poles are connected by wires, so that exactly 6 wires come out of each. How many wires are strung between the poles?

SOLUTION

Returning to the previous notation, we get:

X=9 Y=6

Then we get (9-1)*6=8*6=48

ANSWER: 48

13. SAWING BOARDS AND LOGS

There were several logs. They made X cuts and it turned out for chumps. How many logs were cut?

SOLUTION

When solving, let's make one remark: some problems do not always have a mathematical solution.

Now to the task. When deciding, it must be taken into account that there are more than one logs and when sawing each log, = 1 piece is obtained.

This type of problem is more convenient to solve by the selection method:

Let there be two logs, then the pieces will be 13 + 2 = 15

Take three we get 13+3=16

And here you can see the dependence that the number of cuts and pieces increases in the same way, that is, the number of logs that need to be cut is equal to Y-X

EXAMPLE

There were several logs. We made 13 cuts and got 20 chubachki. How many logs were cut?

SOLUTION

Returning to our reasoning, we can pick up, or you can just 20-13 \u003d 7 means only 7 logs

Answer 7

14 . LOSSED PAGES

Several pages fell out of the book. The first of the dropped pages has the number X, and the number of the last one is written in the same numbers in some other order. How many pages fell out of the book?

SOLUTION

The numbering of pages that have fallen out starts with an odd number and must end with an even number. Therefore, we, knowing that the number of the last dropped out is written in the same digits, that the first dropped out we know its last digit. By permuting the remaining digits and, given that the page numbering must be greater than the first one, we get its number. Knowing the page numbers, you can calculate how many of them fell out, while taking into account that page X also fell out. So from the resulting number we must subtract the number (X-1)

EXAMPLE

Several pages fell out of the book. The first of the dropped pages has the number 387, and the number of the last one is written in the same numbers in some other order. How many pages fell out of the book?

SOLUTION

Based on our reasoning, we get that the number of the last dropped page should end with the number 8. So we have only two options for numbers, these are 378 and 738. 378 does not suit us because it is less than the number of the first dropped page, which means the last dropped out is 738.

738-(387-1)=352

ANSWER: 352

The following should be added: sometimes they ask you to indicate the number of sheets, then the number of pages should be divided in half.

15. FINAL GRADE

At the end of the quarter, Little Johnny wrote down his current marks in singing in a row and put a multiplication sign between some of them. The products of the resulting numbers turned out to be equal to X. What mark does Vovochka have in the singing quarter?

SOLUTION

When solving this type of problem, it must be taken into account that its estimates should be 2,3,4 and 5. Therefore, we need to decompose the number X into factors 2,3,4 and 5. Moreover, the remainder of the expansion should also consist of these numbers.

EXAMPLE1

At the end of the quarter, Little Johnny wrote down his current marks in singing in a row and put a multiplication sign between some of them. The products of the resulting numbers turned out to be equal to 2007. What mark does Vovochka have in the singing quarter?

SOLUTION

Let's factorize the number 2007

We get 2007=3*3*223

So his grades are: 3 3 2 2 3 now find the arithmetic mean of his grades for this set is 2.6 hence his grade is three (greater than 2.5)

ANSWER 3

EXAMPLE 2

At the end of the quarter, Vovochka wrote down all his marks in a row for one of the subjects, there were 5 of them, and put multiplication signs between some of them. The product of the resulting numbers turned out to be 690. What mark does Vovochka get in the quarter in this subject, if the teacher puts only marks 2, 3, 4 and 5 and the final mark in the quarter is the arithmetic average of all current marks, rounded according to the rounding rules? (For example: 2.4 rounds up to two; 3.5 rounds up to 4; and 4.8 rounds up to 5.)

SOLUTION

We factorize 690 so that the remainder of the decomposition consists of the numbers 2 3 4 5

690=3*5*2*23

Hence his scores: 3 5 2 2 3

Let's find the arithmetic mean of these numbers: (3+5+2+2+3)/5=3

This will be his assessment.

ANSWER: 3

16 . MENU

The restaurant menu has X types of salads, Y types of first courses, A types of second courses, and B types of dessert. How many salad, first, second and dessert lunch options can diners at this restaurant choose?

SOLUTION

When solving, we will cut the menu a little: let there be only a salad and then the first options will become (X * Y). Now let's add the second dish, the number of options increases by A times and becomes (X*Y*A). Now let's add dessert. The number of options will increase by B times

Now we get the final answer:

N=X*U*A*B

EXAMPLE

SOLUTION
Based on the above, we get:

N=6*3*5*4=360

ANSWER: 360

17 . WE DIVIDE WITHOUT RESIDUE

In this section, we will consider the tasks on a specific example, for greater clarity.

Since we have a product of consecutive numbers and there are more than 7 of them, then at least one must be divisible by 7. So we have a product, one of the factors of which is divisible by 7, therefore, the whole product is also divisible by seven, which means the remainder of the division will be equal to zero, or for the second problem, the number of factors must be equal to the divisor.

18.TOURISTS

We will also consider this type of tasks with a specific example.

First, let's define what we need to find: route time = ascent + rest + descent

Rest we know, now we need to find the time of ascent and descent

Reading the problem, we see that in both cases (ascent and descent), the time depends as an arithmetic progression, but we still do not know what height the ascent was, although it is not difficult to find:

H=(95-50)15+1=4

We have found the height of the rise, now we will find the rise time as the sum of an arithmetic progression: Tlift = ((2*50+15*(4-1))*4)/2=290 minutes

Similarly, we find, given that now the progression difference is -10. We get Tdesc=((2*60-10(4-1))*4)/2= 180 minutes.

Knowing all the components, you can calculate the total time of the route:

T route = 290 + 180 + 10 = 480 minutes or converting to hours (divided by 60) we get 8 hours.

ANSWER: 8 hours

19. RECTANGLES

There are two types of problems for rectangles: for perimeters and for areas.

To solve such a plan of tasks, it is easy to prove that when splitting any rectangle with two straight cuts, we get four rectangles for which the following relations will always hold:

P1+P2=P3+P4

S1*S2=S3*S4,

where R perimeter , S - square

Based on these relations, we can easily solve the following problems

19.1.Perimeters

SOLUTION

Based on the above, we get

24+16=28+X

X=(24+16)-28=12

ANSWER: 12

19.2 AREAS

The rectangle is divided into four small rectangles by two straight cuts. The areas of three of them, starting from the top left and going clockwise, are 18, 12, and 20. Find the area of ​​the fourth rectangle.

SOLUTION

For the resulting rectangles, the following should be performed:

18*20=12*X

Then X=(18*20)/12=30

ANSWER: 30

20. THERE-HERE

A snail crawls up a tree by A m in a day, and slides down by B m in a night. The height of a tree is C m. In how many days will a snail crawl to the top of a tree for the first time?

SOLUTION

In one day, a snail can rise to a height of (A-B) meters. Since she can climb height A in one day, she must overcome height (C-A) before the last ascent. Based on this, we get that it will rise (C-A) \ (A-B) + 1 (we add one since it rises to height A in one day).

EXAMPLE

SOLUTION

Returning to our reasoning, we get

(10-4)/(4-3)+1=7

REPLY in 7 days

It should be noted that in this way it is possible to solve problems of filling something when something comes in and something flows out.

21. STRAIGHT JUMPS

The grasshopper jumps along the coordinate line in any direction for a unit segment per jump. How many different points on the coordinate line are there that the grasshopper can reach after making X jumps, starting from the origin?

SOLUTION

Suppose that the grasshopper makes all jumps in one direction, then it will hit the point with coordinate X. Now it jumps forward for (X-1) jumps and one back: it hits the point with coordinate (X-2). Considering all his jumps in this way, you can see that he will be at points with coordinates X, (X-2), (X-4), etc. This dependence is nothing more than an arithmetic progression with a differenced\u003d -2 and a1 \u003d X, andan=- X. Then the number of members of this progression is the number of points in which it can be. Let's find them

an=a1+d(n-1)

X=X+d(n-1)

2X=-2(n-1)

n=X+1

EXAMPLE

SOLUTION

Based on the above findings, we get

10+1=11

ANSWER 11 points

TASKS FOR INDEPENDENT SOLUTION:

1. Every second a bacterium divides into two new bacteria. It is known that the entire volume of one glass of bacteria is filled in 1 hour. In how many seconds will the glass be half filled with bacteria?

2. On the stick are marked transverse lines of red, yellow and green. If you cut a stick along the red lines, you get 15 pieces, if along the yellow lines - 5 pieces, and if along the green lines - 7 pieces. How many pieces will you get if you cut a stick along the lines of all three colors?

3. The grasshopper jumps along the coordinate line in any direction by a single segment per jump. The grasshopper starts jumping from the origin. How many different points on the coordinate line are there that the grasshopper can reach after making exactly 11 jumps?

4. There are 40 mushrooms in the basket: mushrooms and milk mushrooms. It is known that among any 17 mushrooms there is at least one camelina, and among any 25 mushrooms at least one mushroom. How many mushrooms are in the basket?

5. Sasha invited Petya to visit, saying that he lives in the seventh entrance in apartment No. 462, but he forgot to say the floor. Approaching the house, Petya discovered that the house had seven floors. What floor does Sasha live on? (On all floors, the number of apartments is the same, the numbers of apartments in the building start from one.)

6. Sasha invited Petya to visit, saying that he lives in the eighth entrance in apartment No. 468, but he forgot to say the floor. Approaching the house, Petya discovered that the house had twelve floors. What floor does Sasha live on? (On all floors, the number of apartments is the same, the numbers of apartments in the building start from one.)

7. Sasha invited Petya to visit, saying that he lives in the twelfth entrance in apartment No. 465, but he forgot to say the floor. Approaching the house, Petya discovered that the house had five floors. What floor does Sasha live on? (On all floors, the number of apartments is the same, the numbers of apartments in the building start from one.)

8. Sasha invited Petya to visit, saying that he lives in the tenth entrance in apartment No. 333, but he forgot to say the floor. Approaching the house, Petya discovered that the house had nine floors. What floor does Sasha live on? (On all floors, the number of apartments is the same, the numbers of apartments in the building start from one.)

9. The trainer advised Andrey to spend 15 minutes on the treadmill on the first day of classes, and on each next lesson to increase the time spent on the treadmill by 7 minutes. How many sessions will Andrey spend on the treadmill for a total of 2 hours and 25 minutes if he follows the advice of the trainer?

10. The doctor prescribed the patient to take the medicine according to the following scheme: on the first day he should take 3 drops, and on each next day - 3 drops more than on the previous one. Having taken 30 drops, he drinks 30 drops of the medicine for another 3 days, and then reduces the intake by 3 drops daily. How many vials of medicine should a patient buy for the entire course of treatment if each contains 20 ml of medicine (which is 250 drops)?

11. The doctor prescribed the patient to take the medicine according to the following scheme: on the first day he should take 20 drops, and on each next day - 3 drops more than on the previous one. After 15 days of taking the patient takes a break of 3 days and continues to take the medicine according to the reverse scheme: on the 19th day he takes the same number of drops as on the 15th day, and then reduces the dose by 3 drops daily until the dosage becomes less than 3 drops per day. How many vials of medicine should a patient buy for the entire course of treatment if each contains 200 drops?

12. The product of ten consecutive numbers is divided by 7. What can be the remainder?

13. In how many ways can two identical red dice, three identical green dice and one blue dice be lined up?

14. A full bucket of water with a volume of 8 liters is poured into a tank with a volume of 38 liters every hour, starting at 12 o'clock. But there is a small gap in the bottom of the tank, and 3 liters flow out of it in an hour. At what point in time (in hours) will the tank be completely filled.

15. What is the smallest number of consecutive numbers that must be taken so that their product is divisible by 7?

16. As a result of the flood, the pit was filled with water up to a level of 2 meters. The construction pump continuously pumps out water, lowering its level by 20 cm per hour. Groundwater, on the contrary, raises the water level in the pit by 5 cm per hour. How many hours of pump operation will the water level in the pit drop to 80 cm?

17. The restaurant menu has 6 types of salads, 3 types of first courses, 5 types of second courses and 4 types of dessert. How many salad, first, second and dessert lunch options can diners at this restaurant choose?

18. An oil company is drilling a well for oil production, which, according to geological exploration, lies at a depth of 3 km. During the working day, drillers go 300 meters deep, but during the night the well “silts up” again, that is, it is filled with soil by 30 meters. How many working days will oil workers drill a well to the depth of oil?

19. What is the smallest number of consecutive numbers that must be taken so that their product is divisible by 9?

20.

for 2 gold coins get 3 silver and one copper;

for 5 silver coins, get 3 gold and one copper.

21. On the surface of the globe, 12 parallels and 22 meridians were drawn with a felt-tip pen. Into how many parts did the drawn lines divide the surface of the globe?

A meridian is an arc of a circle connecting the North and South Poles. A parallel is a circle lying in a plane parallel to the plane of the equator.

22. There are 50 mushrooms in the basket: mushrooms and milk mushrooms. It is known that among any 28 mushrooms there is at least one camelina, and among any 24 mushrooms at least one mushroom. How many mushrooms are in the basket?

23. A group of tourists overcame a mountain pass. They covered the first kilometer of the ascent in 50 minutes, and each next kilometer passed 15 minutes longer than the previous one. The last kilometer before the summit was completed in 95 minutes. After a ten-minute rest at the top, the tourists began their descent, which was more gentle. The first kilometer after the summit was covered in an hour, and each next one is 10 minutes faster than the previous one. How many hours did the group spend on the entire route if the last kilometer of the descent was covered in 10 minutes.

24. There are four gas stations on the ring road: A, B, C and D. The distance between A and B is 35 km, between A and C is 20 km, between C and D is 20 km, between D and A is 30 km (all distances measured along the ring road in the shortest direction). Find the distance between B and C. Give your answer in kilometers.

25. There are four gas stations on the ring road: A, B, C and D. The distance between A and B is 50 km, between A and C is 40 km, between C and D is 25 km, between D and A is 35 km (all distances measured along the ring road in the shortest direction). Find the distance between B and C.

26. There are 25 students in the class. Several of them went to the cinema, 18 people went to the theater, and 12 people went to the cinema and the theater. It is known that three did not go to the cinema or to the theater. How many people in the class went to the movies?

27. According to Moore's empirical law, the average number of transistors on a chip doubles every year. It is known that in 2005 the average number of transistors on a chip was 520 million. Determine how many millions of transistors on a chip were on average in 2003.

28. There are 24 seats in the first row of the cinema hall, and in each next row there are 2 more than in the previous one. How many seats are in the eighth row?

29. On the stick are marked transverse lines of red, yellow and green. If you cut a stick along the red lines, you get 5 pieces, if along the yellow lines - 7 pieces, and if along the green lines - 11 pieces. How many pieces will you get if you cut a stick along the lines of all three colors?

30. In a home appliance store, sales of refrigerators are seasonal. In January, 10 refrigerators were sold, and in the next three months, 10 refrigerators were sold. Since May, sales have increased by 15 units compared to the previous month. Since September, sales began to decrease by 15 refrigerators every month compared to the previous month. How many refrigerators did the store sell in a year?

31. In the exchange office, you can perform one of two operations:

1) for 3 gold coins get 4 silver and one copper;

2) for 6 silver coins, get 4 gold and one copper.

Nikola had only silver coins. After visiting the exchange office, he had fewer silver coins, no gold coins, but 35 copper coins appeared. By how much did Nikola's number of silver coins decrease?

32. Sasha invited Petya to visit, saying that he lives in the seventh entrance in apartment No. 462, but he forgot to say the floor. Approaching the house, Petya discovered that the house had seven floors. What floor does Sasha live on? (The number of apartments on each floor is the same, the numbers of apartments in the building start from one.)

33. All entrances of the house have the same number of floors, and each floor has the same number of apartments. At the same time, the number of floors in the house is greater than the number of apartments per floor, the number of apartments per floor is greater than the number of entrances, and the number of entrances is more than one. How many floors are there in a building if there are 110 apartments in total?

34. The grasshopper jumps along the coordinate line in any direction for a unit segment per jump. How many different points on the coordinate line are there that the grasshopper can reach after making exactly 6 jumps, starting from the origin?

35. There are 40 mushrooms in the basket: mushrooms and milk mushrooms. It is known that among any 17 mushrooms there is at least one camelina, and among any 25 mushrooms at least one mushroom. How many mushrooms are in the basket?

36. There are 25 mushrooms in the basket: mushrooms and milk mushrooms. It is known that among any 11 mushrooms there is at least one camelina, and among any 16 mushrooms at least one mushroom. How many mushrooms are in the basket?

37. There are 30 mushrooms in the basket: mushrooms and milk mushrooms. It is known that among any 12 mushrooms there is at least one camelina, and among any 20 mushrooms at least one mushroom. How many mushrooms are in the basket?

38. On the globe, 17 parallels (including the equator) and 24 meridians were drawn with a felt-tip pen. Into how many parts do the lines drawn divide the surface of the globe?

39. A snail crawls 4 m up a tree in a day, and slides 3 m in a night. The height of a tree is 10 m. In how many days will a snail crawl to the top of a tree for the first time?

40. A snail crawls 4 m up a tree in a day, and slides 1 m in a night. The height of a tree is 13 m. How many days does it take for a snail to crawl to the top of a tree for the first time?

41. The owner agreed with the workers that they would dig a well for him on the following terms: for the first meter he would pay them 4,200 rubles, and for each next meter - 1,300 rubles more than for the previous one. How much money will the owner have to pay the workers if they dig a well 11 meters deep?

42. The owner agreed with the workers that they were digging a well on the following terms: for the first meter he would pay them 3,500 rubles, and for each next meter - 1,600 rubles more than for the previous one. How much money will the owner have to pay the workers if they dig a well 9 meters deep?

43. There are 45 mushrooms in the basket: mushrooms and milk mushrooms. It is known that among any 23 mushrooms there is at least one camelina, and among any 24 mushrooms at least one mushroom. How many mushrooms are in the basket?

44. There are 25 mushrooms in the basket: mushrooms and milk mushrooms. It is known that among any 11 mushrooms there is at least one camelina, and among any 16 mushrooms at least one mushroom. How many mushrooms are in the basket?

45. The list of tasks of the quiz consisted of 25 questions. For each correct answer, the student received 7 points, for an incorrect answer, 10 points were deducted from him, and if there was no answer, they were given 0 points. How many correct answers were given by the student who scored 42 points, if it is known that he was wrong at least once?

46. On the stick are marked transverse lines of red, yellow and green. If you saw the stick along the red lines, you get 5 pieces, if along the yellow lines - 7 pieces, and if along the green lines - 11 pieces. How many pieces will you get if you cut a stick along the lines of all three colors?

47. A snail crawls up a tree 2 m in a day, and slides down 1 m in a night. The height of the tree is 11 m. How many days will it take for a snail to crawl from the base to the top of the tree?

48. A snail crawls 4 m up a tree in a day, and slides 2 m in a night. The height of a tree is 14 m. How many days will it take for a snail to crawl from the base to the top of the tree?

49. The rectangle is divided into four smaller rectangles by two straight cuts. The perimeters of three of them, starting from the top left and proceeding clockwise, are 24, 28 and 16. Find the perimeter of the fourth rectangle.

50. In the exchange office, you can perform one of two operations:

1) for 2 gold coins get 3 silver and one copper;

2) for 5 silver coins, get 3 gold and one copper.

Nicholas had only silver coins. After several visits to the exchange office, he had fewer silver coins, no gold coins, but 50 copper coins appeared. By how much did Nicholas's number of silver coins decrease?

51. The rectangle is divided into four smaller rectangles by two straight cuts. The perimeters of three of them, starting from the top left and proceeding clockwise, are 24, 28 and 16. Find the perimeter of the fourth rectangle.

52. In the exchange office, you can perform one of two operations:

1) for 4 gold coins get 5 silver and one copper;

2) for 7 silver coins, get 5 gold and one copper.

Nicholas had only silver coins. After several visits to the exchange office, he had fewer silver coins, no gold coins, but 90 copper coins appeared. By how much did Nicholas's number of silver coins decrease?

53. All entrances of the house have the same number of floors, and each floor has the same number of apartments. At the same time, the number of entrances of the house is less than the number of apartments per floor, the number of apartments per floor is less than the number of floors, the number of entrances is more than one, and the number of floors is not more than 24. How many floors are there in a house if it has only 156 apartments?

54. V There are 26 students in the class. Several of them listen to rock, 14 people listen to rap, and only three listen to both rock and rap. It is known that four do not listen to either rock or rap. How many people in the class listen to rock?

55. V There are 35 fish in the cage: perches and roaches. It is known that among any 21 fish there is at least one roach, and among any 16 fish there is at least one perch. How many roaches are in the garden?

56. On the surface of the globe, 30 parallels and 24 meridians were drawn with a marker. Into how many parts did the drawn lines divide the surface of the globe? (a meridian is an arc of a circle connecting the North and South poles, and a parallel is the boundary of a section of a globe by a plane parallel to the plane of the equator).

57. V prehistoric exchange office could do one of two things:
- For 2 skins of a cave lion, get 5 skins of a tiger and 1 skin of a wild boar;
- For 7 tiger skins, get 2 cave lion skins and 1 wild boar skin.
Un, the son of the Bull, had only the skins of a tiger. After several visits to the exchange office, the tiger skins did not increase, the cave lion skins did not appear, but 80 wild boar skins appeared. By how much did Un, the Bull's son, have the number of tiger skins reduced in the end?

58. V military unit 32103 has 3 types of salad, 2 types of first course, 3 types of second course and a choice of compote or tea. How many lunch options, which must consist of one salad, one first course, one second course and one drink, can the soldiers of this military unit choose?

59. A snail crawls 5 meters up a tree during the day, and slides down 3 meters during the night. The height of the tree is 17 meters. On what day will the snail crawl to the top of the tree for the first time?

60. In how many ways can three identical yellow cubes, one blue cube and one green cube be placed in a row?

61. The product of sixteen consecutive natural numbers is divided by 11. What can be the remainder of the division?

62. Every minute a bacterium divides into two new bacteria. It is known that bacteria fill the entire volume of a three-liter jar in 4 hours. How many seconds does it take bacteria to fill a quarter of a jar?

63. The list of tasks of the quiz consisted of 36 questions. For each correct answer, the student received 5 points, for an incorrect answer, 11 points were deducted from him, and if there was no answer, they were given 0 points. How many correct answers were given by the student who scored 75 points, if it is known that he was wrong at least once?

64. The grasshopper jumps along a straight road, the length of one jump is 1 cm. First, he jumps 11 jumps forward, then 3 back, then again 11 jumps and then 3 jumps back, and so on, how many jumps he will make by the time when he first finds himself at a distance of 100 cm from the start.

65. On the stick are marked transverse lines of red, yellow and green. If you cut a stick along the red lines, you get 7 pieces, if along the yellow lines - 13 pieces, and if along the green lines - 5 pieces. How many pieces will you get if you cut a stick along the lines of all three colors?

66. V The exchange office can perform one of two operations:
for 2 gold coins get 3 silver and one copper;
for 5 silver coins, get 3 gold and one copper.
Nicholas had only silver coins. After several visits to the exchange office, he had fewer silver coins, no gold coins, but 50 copper coins appeared. By how much did Nicholas's number of silver coins decrease?

67. The rectangle is divided into four smaller rectangles by two straight cuts.
The perimeters of three of them, starting from the top left and proceeding clockwise, are 24, 28 and 16. Find the perimeter of the fourth rectangle.

68. V The exchange office can perform one of two operations:
1) for 4 gold coins get 5 silver and one copper;
2) for 7 silver coins, get 5 gold and one copper.
Nikola had only silver coins. After visiting the exchange office, he had fewer silver coins, no gold coins, but 90 copper coins appeared. By how much did the number of silver coins decrease?

69. A snail crawls 4 m up a tree in a day, and slides 2 m in a night. The height of a tree is 12 m. How many days will it take for a snail to crawl from the base to the top of the tree?

70. The list of tasks of the quiz consisted of 32 questions. For each correct answer, the student receives 5 points. 9 points were written off for the wrong answer, 0 points were given if there was no answer.
How many correct answers did the student who scored 75 points give if he was wrong at least 2 times?

71. The list of tasks of the quiz consisted of 25 questions. For each correct answer, the student received 7 points, for an incorrect answer, 10 points were deducted from him, and if there was no answer, they were given 0 points. How many correct answers were given by the student who scored 42 points, if it is known that he was wrong at least once?

72. The owner agreed with the workers that they would dig a well for him on the following terms: for the first meter he would pay them 4,200 rubles, and for each next meter - 1,300 rubles more than for the previous one. How many rubles will the owner have to pay to the workers if they dig a well 11 meters deep?

73. The rectangle is divided into four small rectangles by two straight cuts. The areas of three of them, starting from the top left and going clockwise, are 18, 12, and 20. Find the area of ​​the fourth rectangle.

74. The rectangle is divided into four small rectangles by two straight cuts. The areas of three of them, starting from the top left and going clockwise, are 12, 18, and 30. Find the area of ​​the fourth rectangle.

75. V The table has three columns and several rows. V each cell of the table was placed according to a natural number so that the sum of all the numbers in the first column is 85, in the second - 77, in the third - 71, and the sum of the numbers in each row is greater than 12, but less than 15. How many rows are there in the table?

76. The grasshopper jumps along the coordinate line in any direction for a unit segment per jump. How many different points on the coordinate line are there that the grasshopper can reach after making 10 jumps, starting from the origin?

77. Sasha invited Petya to visit, saying that he lives in the seventh entrance in apartment No. 462, but he forgot to say the floor. Approaching the house, Petya discovered that the house had seven floors. What floor does Sasha live on? (On all floors, the number of apartments is the same, the numbers of apartments in the building start from one.)

78. V The exchange office can perform one of two operations:
for 2 gold coins get 3 silver and one copper;
for 7 silver coins get 3 gold and one copper.
Nicholas had only silver coins. After the exchange office, he did not have gold coins, but 20 copper coins appeared. By how much did Nicholas's number of silver coins decrease?

79. The grasshopper jumps along the coordinate line in any direction for a unit segment per jump. How many different points on the coordinate line are there that the grasshopper can reach after making 11 jumps, starting from the origin?

80. There are four gas stations on the ring road: A, B, C and D. The distance between A and B - 35 km, between A and B - 20 km, between B and G - 20 km, between G and A - 30 km (all distances are measured along the ring road in the shortest arc). Find the distance (in kilometers) between B and V.

81. V The exchange office can perform one of two operations:
for 4 gold coins get 5 silver and one copper;
for 7 silver coins get 5 gold and one copper.
Nicholas had only silver coins. After the exchange office, he had fewer silver coins, there were no gold coins, but 90 copper coins appeared. By how much did Nicholas's number of silver coins decrease?

82. A grasshopper jumps along a coordinate line in any direction for a unit segment per jump. How many points on the coordinate line are there that the grasshopper can reach after making exactly 8 jumps, starting from the origin?

83. V The exchange office can perform one of two operations:
for 5 gold coins get 4 silver and one copper;
For 10 silver coins, get 7 gold and one copper.
Nicholas had only silver coins. After the exchange office, he had fewer silver coins, no gold coins, but 60 copper coins appeared. By how much did Nicholas's number of silver coins decrease?

84. V The exchange office can perform one of two operations:
for 5 gold coins get 6 silver and one copper;
for 8 silver coins get 6 gold and one copper.
Nicholas had only silver coins. After the exchange office, he had fewer silver coins, no gold coins, but 55 copper coins appeared. By how much did Nicholas's number of silver coins decrease?

85. All entrances of the house have the same number of floors, and all floors have the same number of apartments. At the same time, the number of floors in the house is greater than the number of apartments per floor, the number of apartments per floor is greater than the number of entrances, and the number of entrances is more than one. How many floors are there in a building if there are 105 apartments in total?

86. V The exchange office can perform one of two operations:
1) for 3 gold coins get 4 silver and one copper;
2) for 7 silver coins, get 4 gold and one copper.
Nikola had only silver coins. After visiting the exchange office, he had fewer silver coins, no gold coins, but 42 copper coins appeared. By how much did Nikola's number of silver coins decrease?

ANSWERS

Task No. 20 of the Unified State Examination in mathematics contains a quick wit task. The tasks in this section are more intuitive than in task 19 of the Unified State Examination, but nevertheless they are quite difficult for an ordinary student. So, let's move on to the consideration of typical options.

Analysis of typical options for assignments No. 20 USE in mathematics of a basic level

The first version of the task (demo version 2018)

  • for 2 gold coins get 3 silver and one copper;
  • for 5 silver coins, get 3 gold and one copper.

Nicholas had only silver coins. After several visits to the exchange office, he had fewer silver coins, no gold coins, but 50 copper coins appeared. By how much did Nicholas's number of silver coins decrease?

Execution algorithm:
  1. Enter symbols.
  2. Record task data using legend.
  3. Logically identify the unknown.
Solution:

According to the condition, no gold coins appeared, which means that all the gold coins received after the second operation were exchanged by Nikolai using the first operation. Gold coins can only be changed in 2 pieces, therefore, there were an even number of second transactions.

Let's introduce the notation, let there be 2n second operations (the number is always even).

If we apply the second operation, we get:

All gold coins were exchanged during the first operation. For one operation, you can exchange 2 gold coins at once, which means that (3 · 2n)/2 = 3 n will be completed in total. That is

3 2n gold coins were exchanged for 3 3n silver + 3n copper.

Or after conversion:

Let's compare the results of the first and second operations:

5 x 2n silver was exchanged for 3 x 2n gold + 2n copper.

3 2n gold exchanged for 9n silver + 3n copper

5 2n silver exchanged for 9n silver + 3n copper + 2n copper

10n silver exchanged for 9n silver + 5n copper

If, having exchanged 10 n silver coins, we get 9 n silver coins, then Nikolai's number of silver coins has decreased by n. It can be seen from the last expression that Nikolai received 5n copper coins, and according to the condition, 50 copper coins appeared, that is, 5n = 50.

The second version of the task

Masha and the Bear ate 100 cookies and a jar of jam, starting and finishing at the same time. At first, Masha ate jam, and the Bear ate cookies, but at some point they changed. The bear eats both three times faster than Masha. How many cookies did the Bear eat if they ate the same amount of jam?

Execution algorithm:
  1. Compare results.
  2. Find the unknown.
Solution:
  1. Since both Masha and the Bear ate the jam equally, and at the same time the Bear ate the jam 3 times faster, Masha ate the jam (her half) 3 times longer than the Bear (the same half).
  2. Then it turns out that the Bear ate cookies 3 times longer than Masha and, moreover, ate them 3 times faster, that is, for one cookie Masha ate, there were 3∙3=9 cookies eaten by the Bear.
  3. These cookies add up to 1+9=10 and there are exactly 100:10 = 10 such sums in 100 cookies.
  4. So Masha ate 10 cookies, and the Bear 9∙10=90.

The third version of the task

Masha and the Bear ate 51 cookies and a jar of jam, starting and finishing at the same time. At first, Masha ate jam, and the Bear ate cookies, but at some point they changed. The bear eats both four times faster than Masha. How many cookies did the Bear eat if they ate the same amount of jam?

Execution algorithm:
  1. Determine who and how many times longer ate cookies.
  2. Determine who and how many times longer ate jam.
  3. Compare results.
  4. Find the unknown.
Solution:
  1. Since both Masha and the Bear ate the jam equally, and at the same time the Bear ate the jam 4 times faster, Masha ate the jam (her half) 4 times longer than the Bear (the same half).
  2. Then it turns out that the Bear ate cookies 4 times longer than Masha and, moreover, ate them 4 times faster, that is, for one cookie Masha ate, there were 4∙4=16 cookies eaten by the Bear.
  3. These cookies add up to 1+16=17 and there are exactly 51:17 = 3 such sums in 51 cookies.
  4. So, Masha ate 3 cookies, and the Bear 3∙16=48.

The fourth option

If each of the two factors were increased by 1, their product would increase by 11. In fact, each of the two factors was increased by 2. By how much did the product increase?

Execution algorithm:
  1. Enter symbols.
  2. Convert the resulting expression.
  3. Find the unknown.
Solution:

When these factors are increased by 1, their product increases by 11, that is,

Now, similarly, we calculate how much the product will increase if the factors are increased by 2 and substitute the already known a + b = 10:

Fifth option

If each of the two factors were increased by 1, their product would increase by 3. In fact, each of the two factors was increased by 5. By how much did the product increase?

Execution algorithm:
  1. Enter symbols.
  2. Write the first condition using the notation.
  3. Convert the resulting expression.
  4. Write down the second condition using symbols.
  5. Convert the resulting expression.
  6. Find the unknown.
Solution:

Let the first factor be equal to a, and the second b, their product is equal to ab.

When these factors are increased by 1, their product increases by 3, that is,

We transfer the product ab to the left side with the opposite sign and open the brackets by multiplying.

Now, similarly, we calculate how much the product will increase if the factors are increased by 5 and substitute the already known a + b = 2:

Variant of the twentieth task 2017

The rectangle is divided into four smaller rectangles by two straight line segments. The perimeters of three of them, starting from the top left and proceeding clockwise, are 24, 28 and 16. Find the perimeter of the fourth rectangle.

Let's redraw the rectangle in a form convenient for us:

Now let's write equations using the formula for the perimeter of a rectangle:

Variant of the twentieth task of 2019 (1)

The list of tasks of the quiz consisted of 25 questions. For each correct answer, the student received 7 points, for an incorrect answer, 10 points were deducted from him, and if there was no answer, they were given 0 points. How many correct answers were given by the student who scored 42 points, if it is known that he was wrong at least once?

Execution algorithm
  1. We make combinations of correct and incorrect answers and determine the number of points in them, for example: 1) 1 right + 1 wrong \u003d 7–10 \u003d -3 points; 2) 2 right + 1 wrong = 2 7–10 = 4 points, etc.
  2. Of the points for the right answers and the points for their combinations, we “gain” 42 points. We count the number of questions that were asked at the same time.
  3. The remaining difference between the number of questions received and the given 25 questions is defined as those that were not answered.
  4. Let's check the result.
Solution:

Let's introduce the notation: right answer - 1P, wrong answer - 1H.

We set combinations and determine the number of points that will be awarded in this case:

1P=7 points

1P+1N=7–10=–3 b.

2P+1H=2 7–10=4 b.

3P+1N=3 7–10=11 b.

Let's summarize the points that can be obtained in this case: 7+ (–3)+4+11=19. This is clearly not enough. And you can definitely add 11 more: 19+11=30. To "get" up to 42 points, you must further add 12 points, which are scored by triple occurrence of 4 points. In general, we get:

7+(–3)+4+11+11+3 4=42.

We write the resulting combination of terms in the form of answers:

1P+(1P+1N)+(2P+1N)+(3P+1N)+(3P+1N)+3 (2P+1N)=1P+1P+1N+2P+1N+3P+1N+3P+ 1N + 6P + 3N \u003d 16P + 7N (answers).

16+7=23 answers. 25–23=2 responses for which 0 points were received, i.e. these are questions left unanswered.

So, according to our calculations, 16 correct answers were given.

Let's check it out:

16 answers for 7 b. + 7 answers for (-10) b. + 2 answers for 0 b. = 16 7–7 10+2 0=112–70+0=42 (points).

Variant of the twentieth task of 2019 (2)

The table has three columns and several rows. A natural number was written in each cell of the table so that the sum of all the numbers in the first column is 103, in the second - 97, in the third - 93, and the sum of the numbers in each row is greater than 21, but less than 24. How many rows are there in the table?

Execution algorithm
  1. Find the total for all the numbers in the table (by adding the sums for each of the 3 columns).
  2. We determine the range of valid values ​​for the sums of numbers in each line.
  3. Dividing the total amount first by the smallest sum of numbers in each line, and then by the largest, we get the required number of lines.
Solution:

The total sum of the numbers in the table is: 103+97+93=293.

Since by condition the sums of numbers in each row are >21, but<24, то кол-во строк X может быть равным меньше, чем 293:21≈13,95, и больше, чем 293:24≈12,21. Т.е.: 12,21 < X < 13,95. Единственное целое число в полученном диапазоне – 13. Значит, искомое кол-во строк равно 13.

Variant of the twentieth task of 2019 (3)

There are only eighteen apartments in the house, numbered from 1 to 18. At least one and no more than three people live in each apartment. A total of 15 people live in apartments from 1st to 13th inclusive, and a total of 20 people live in apartments from 11th to 18th inclusive. How many people live in this house?

Execution algorithm
  1. We determine the maximum number of people living in apartments 11–13 using data on how many people live in apartments 1–13.
  2. We find the minimum number of residents of apartments 11–13, taking into account the data on those living in apartments 11–18.
  3. Comparing the data obtained in paragraphs 1–2, we obtain the exact number of residents of these apartments Nos. 11–13.
  4. We find the number of people living in apartments 1-10th and 14-18th.
  5. We calculate the total number of residents of the house.
Solution:

In the first 13 apartments (from the 1st to the 13th) there are 15 people. This means that 1 person lives in 11 apartments plus 2 people in 2 apartments (11 1+2 2=15). Consequently, at least 3 and no more than 5 (1+2+2) people live in apartments 11–13 (i.e., in 3 apartments).

The second 8 apartments (11th to 18th) are home to 20 people. At the same time, from the 14th to the 18th apartments (i.e., in 5 apartments), more than 5 3 = 15 people cannot live. And consequently, no less than 20–15=5 people live in apartments 11-13.

Those. on the one hand, no more than 5 people should live in apartments 11-13, and on the other, at least 5 people. Conclusion: exactly 5 people live in these apartments, because. there are no other valid values ​​for both cases.

Then we get: 15–5=10 people live in apartments 1–10, 20–5=15 people live in apartments 14–18. In total, the house is inhabited by: 10+5+15=30 people.

Variant of the twentieth task of 2019 (4)

In the exchange office, you can perform one of two operations:

  • for 4 gold coins get 5 silver and one copper;
  • for 7 silver coins get 5 gold and one copper.

Nicholas had only silver coins. After several visits to the exchange office, he had fewer silver coins, no gold coins, but 45 copper coins appeared. By how much did Nicholas's number of silver coins decrease?

Execution algorithm
  1. We determine the number of silver coins that Nikolai needs to make a double exchange so that he does not have gold coins. Double exchange is the exchange of first silver coins for gold and copper, and then gold for silver and copper.
  2. We determine the number of different coins that Nikolai will have as a result of 1 double exchange.
  3. We calculate the number of double exchanges that must be made in order for 45 copper coins to appear.
  4. We find the number of silver coins that Nikolai had to have initially in order to make the required number of exchanges, and which he received as a result of all exchanges.
  5. We determine the desired difference.
Solution:

Nikolay must make the 1st exchange according to the 2nd scheme, because he only has silver coins. In order for him not to have gold coins as a result, you need to find the minimum multiple of 5 gold coins that he will receive, and 4 gold coins that can be accepted from him at a time in full (without a remainder). This is the number 20.

Accordingly, in order to get 20 gold coins, Nikolai must have 20:5 = 4 sets of 7 silver coins. So, initially he should have 4·7=28 of them. And at the same time, Nikolai also receives 1 4 = 4 copper coins.

Making an exchange, Nikolay gives 20:4=5 sets of gold medals. In return, he receives 5 x 5 = 25 silver coins and 1 x 5 = 5 copper coins.

Thus, as a result of one exchange, Nikolai will have 25 silver coins and 4 + 5 = 9 copper coins. Since in the end Nicholas had 45 copper coins, it means that 45:9=5 double exchanges were made.

If as a result of 1 double exchange Nikolay had 25 silver coins, then after 5 such exchanges he will have 25 5 = 125 pieces. And initially he had to have 28 5 = 140 silver coins for this. Consequently, their number at Nikolai decreased by 140–125=15 pieces.

Variant of the twentieth task of 2019 (5)

All entrances of the house have the same number of floors, and all floors have the same number of apartments. At the same time, the number of floors in the house is greater than the number of apartments per floor, the number of apartments per floor is greater than the number of entrances, and the number of entrances is more than one. How many floors are there in a building if there are 357 apartments in total?

Execution algorithm
  1. We define the equation for determining the number of apartments in the house in total through the parameters stated in the condition (i.e. through the number of apartments on the floor, etc.).
  2. We factorize 357.
  3. We find the correspondence of the obtained multipliers to specific parameters, proceeding from the condition of which of the parameters is greater or less than the others.
Solution:

Because on all floors the same number of apartments (X), on all entrances the same number of floors (Y), then denoting the number of entrances through Z, we can write: 357=X·Y·Z.

We decompose 357 into prime factors. We get: 357=3 7 17 1. And this is the only scheduling option. Because Y>X>Z>1, then we do not take into account the unit in the layout and determine that Z=3, X=7, Y=17.

Since the number of floors was indicated by Y, the desired number is 17.

Variant of the twentieth task of 2019 (6)

Of the ten countries, seven have signed a friendship treaty with exactly three countries, and each of the remaining three with exactly seven. How many contracts were signed in total?

Execution algorithm
  1. We count the number of agreements signed by 7 countries.
  2. We determine the number of agreements signed by the 3 remaining countries.
  3. Find the total number of signed contracts. We divide it by 2, because bilateral agreements.
Solution:

The first 7 countries have signed agreements with 3 countries, i.е. 7 3 = 21 signatures were put on these contracts. Similarly, the remaining 3 countries, when drawing up contracts with 7 countries, put 3·7=21 signatures. This means that 21+21=42 signatures were put in total.

Because Since all contracts are bilateral, this means that each of them has 2 signatures. Consequently, there are half as many contracts as there are signatures, i.е. 42:2=21 contract.

Variant of the twentieth task of 2019 (7)

On the surface of the globe, 13 parallels and 25 meridians were drawn with a felt-tip pen. Into how many parts did the drawn lines divide the surface of the globe?

A meridian is an arc of a circle that connects the North and South Poles. A parallel is a circle lying in a plane parallel to the plane of the equator.

Execution algorithm
  1. We prove that the parallels divide the globe into 13 + 1 parts.
  2. We prove that the meridians divide the globe into 25 parts.
  3. We determine the number of parts into which the globe is divided as a whole, as a product of the found numbers.
Solution:

If any parallel is a circle, then it is a closed line. And this means that the 1st parallel divides the globe into 2 parts. Further, the 2nd parallel provides a division into 3 parts, the 3rd - into 4, etc. As a result, 13 parallels will divide the globe into 13 + 1 = 14 parts.

The meridian is an arc of a circle connecting the poles, i.e. it is not a closed line and does not divide the globe into parts. But 2 meridians are already dividing, i.e. 2 meridians provide a division into 2 parts, then the 3rd meridian adds the 3rd part, the 4th - the 5th part, etc. So, ultimately, 25 meridians create 25 parts on the globe.

The total number of parts on the globe is: 14 25=350 parts.

Variant of the twentieth task of 2019 (8)

There are 30 mushrooms in the basket: mushrooms and milk mushrooms. It is known that among any 12 mushrooms there is at least one camelina, and among any 20 mushrooms there is at least one mushroom. How many mushrooms are in the basket?

Execution algorithm
  1. We determine the number of milk mushrooms among 12 mushrooms and mushrooms among 20 mushrooms.
  2. We prove that there is only one correct number representing the number of saffron milk caps. We fix it in the answer.
Solution:

If there is at least 1 mushroom among 12 mushrooms, then there are no more than 11 mushrooms. If there is at least 1 mushroom among 20 mushrooms, then there are no more than 19 mushrooms.

This means that if there cannot be more than 11 milk mushrooms, then there cannot be less than 30–11=19 mushrooms. Those. there are no more than 19 mushrooms on one side, and at least 19 on the other. Therefore, there can only be exactly 19 mushrooms.

Variant of the twentieth task of 2019 (9)

If each of the two factors were increased by 1, then their product would increase by 3. By how much will the product of these factors increase if each of them is increased by 5?

Execution algorithm
  1. We introduce the notation for the multipliers. This will allow us to express the original product (before increasing the factors).
  2. We make an equation for the situation when the factors are increased by 1. We perform transformations. We get a new expression that displays the relationship between the original factors.
  3. We make an equation for the situation when the factors are increased by 5. We perform transformations. We introduce the expression obtained in paragraph 2 into the equation, we find the desired difference.
Solution:

Let the 1st factor be equal to x, the 2nd to y. Then their product is xy.

After the multipliers are increased by 1, we get:

(x+1)(y+1)=xy+3

xy + y + x + 1 = xy +3

After increasing the multipliers by 5, we have:

(x + 5) (y + 5) \u003d xy + N, where N is the desired difference in products.

We perform transformations:

xy+5y+5x+25=xy+N

N= xy + 5y + 5x + 25- xy

Because above it is already defined that x + y \u003d 2, then we get:

Variant of the twentieth task of 2019 (10)

Sasha invited Petya to visit, saying that he lives in the seventh entrance in apartment No. 462, but he forgot to say the floor. Approaching the house, Petya discovered that the house had seven floors. What floor does Sasha live on? (On all floors, the number of apartments is the same, the numbering of apartments in the building starts from one.)

Execution algorithm
  1. By the selection method, we determine the number of apartments on the site. This number should be such that the apartment number is greater than the number of apartments in 6 entrances, but less than the number of apartments in 7.
  2. We determine the number of apartments in 6 entrances. We subtract this number from 462 and divide by the number of apartments on the site. So we find out the desired floor number. Note: 1) if an integer is received, then the desired floor number is 1 more than the calculated value; 2) if a fractional number is received, then the floor number will be the result rounded up.
Solution:

We are looking for the number of apartments on the site, checking number by number.

Suppose this number is 3. Then we get that there are 7 6 3 = 126 apartments in 7 entrances on 6 floors,

and in 7 entrances on 7 floors 7 7 3 = 147 apartments.

Apartment #462 definitely does not fall into the range of apartments #126–147.

Similarly, checking the numbers 4, 5, etc., we arrive at the number 10. Let's prove that it is suitable:

in 7 entrances on 6 floors there are 7 6 10=420 apartments,

in 7 entrances on 7 floors: 7 7 10=490 apartments. Since 420<462<490, то условие задания выполнено.

In order to get to apartment No. 462, you need to pass by 462–420=42 apartments. Because there are 10 apartments on each site, then 42:10 = 4.2 floors must be overcome for this. 4.2 means that you need to go through 4 floors completely and go up to the 5th. Thus, the desired floor is the 5th.